Strengthen with Necessary Premise Questions - - Question 9

Despite improvements in treatment for asthma, the death rate from this disease has doubled during the past decade fro...

Ashley-Tien June 21, 2018

Process of Elimination

I was stuck between C and E; how do I eliminate C? If bronchial inhalers were safe, wouldn't that also destroy the conclusion?

Replies
Create a free account to read and take part in forum discussions.

Already have an account? log in

Christopher June 21, 2018

@Ashley-Tien, since this is asking for the assumption which will allow the conclusion to be drawn, you can determine that this is a strengthen with necessary premise question. The thing to look for with these is not only the answer that would allow you to reach the conclusion but also the answer that would weaken the argument if negated.

So (C) could complete the argument, but if you negate it, it doesn't modify the argument. However, if you negate (E) it provides an alternate explanation and thus weakens the argument.

Does that help?

Ohemaa June 25, 2018

Can you explain why C is incorrect thoroughly?

Mehran June 27, 2018

Hey @Ohemaa, it may help to take a step back and see the big picture on this question. The stimulus basically says, here are two possible reasons the death rate for asthma has doubled in the past 10 years. It then rules out those two reasons and concludes that a third reason *must* be the cause of this doubled death rate.

The assumption here is that the universe of possible reasons for the doubled death rate consists of just three possible causes. That way, if two are ruled out, then the third must be the explanation.

This may help, in addition to the negation provided for you in our explanation for answer choice (C), to see why this answer is irrelevant and thus incorrect.

Hope this helps. Please let us know if you have any additional questions.